If X is the set of prime...

This topic has expert replies
Moderator
Posts: 2205
Joined: Sun Oct 15, 2017 1:50 pm
Followed by:6 members

If X is the set of prime...

by BTGmoderatorLU » Thu Oct 19, 2017 7:12 am
If X is the set of prime single-digit numbers and Y is a set containing each of the numbers in set X raised to the power of 2, how much greater is the median of set Y than the median of set X?

A. 2
B. 4
C. 9
D. 13
E. 17

The OA is D.

Why D is the correct answer? Can any expert help me with this PS question please? Thanks.

User avatar
Legendary Member
Posts: 2663
Joined: Wed Jan 14, 2015 8:25 am
Location: Boston, MA
Thanked: 1153 times
Followed by:128 members
GMAT Score:770

by DavidG@VeritasPrep » Thu Oct 19, 2017 7:40 am
LUANDATO wrote:If X is the set of prime single-digit numbers and Y is a set containing each of the numbers in set X raised to the power of 2, how much greater is the median of set Y than the median of set X?

A. 2
B. 4
C. 9
D. 13
E. 17

The OA is D.

Why D is the correct answer? Can any expert help me with this PS question please? Thanks.
Set X [2, 3, 5, 7] Median = (3+5)/2 = 4
Set Y [2^2, 3^2, 5^2, 7^2] or [4, 9, 25, 49] Median = (9+25)/2 = 17
17 - 4 = 13. The answer is D
Veritas Prep | GMAT Instructor

Veritas Prep Reviews
Save $100 off any live Veritas Prep GMAT Course

GMAT/MBA Expert

User avatar
GMAT Instructor
Posts: 7223
Joined: Sat Apr 25, 2015 10:56 am
Location: Los Angeles, CA
Thanked: 43 times
Followed by:29 members

by Scott@TargetTestPrep » Fri Nov 22, 2019 11:53 am
BTGmoderatorLU wrote:If X is the set of prime single-digit numbers and Y is a set containing each of the numbers in set X raised to the power of 2, how much greater is the median of set Y than the median of set X?

A. 2
B. 4
C. 9
D. 13
E. 17

The OA is D.

Why D is the correct answer? Can any expert help me with this PS question please? Thanks.
We see that X = {2, 3, 5, 7} and Y = {4, 9, 25 49}. Therefore, the median of set X is (3 + 5)/2 = 4, and that of set Y is (9 + 25)/2 = 17. So the median of set Y is 17 - 4 = 13 greater than the median of set X.

Answer: D

Scott Woodbury-Stewart
Founder and CEO
[email protected]

Image

See why Target Test Prep is rated 5 out of 5 stars on BEAT the GMAT. Read our reviews

ImageImage